Đến nội dung

Hình ảnh

$\sum_{i=1}^n x_i^2>\frac{1}{4}(\sum_{j=1}^n \frac{1}{j})$


  • Please log in to reply
Chưa có bài trả lời

#1
Minhnguyenthe333

Minhnguyenthe333

    Trung úy

  • Thành viên
  • 804 Bài viết

Cho dãy giảm $n$ số dương $x_1,x_2,x_3,...,x_n$ thỏa mãn điều kiện $x_1+x_2+....+x_k\geqslant \sqrt{k}$ $(n\geqslant k)$.Chứng minh rằng: $x_1^2+x_2^2+...+x_n^2>\frac{1}{4}(1+\frac{1}{2}+...+\frac{1}{n})$






1 người đang xem chủ đề

0 thành viên, 1 khách, 0 thành viên ẩn danh